Đến nội dung

MrMATH

MrMATH

Đăng ký: 26-12-2004
Offline Đăng nhập: 11-05-2022 - 19:13
****-

#39805 Bất đẳng thức thuần nhất

Gửi bởi MrMATH trong 28-10-2005 - 21:35

Rõ ràng là không rồi, bởi vì khi cho k tiến tới o em đã dùng 1 phép giới hạn, cái sự lấy giới hạn này thường được dùng để dự đoán các điểm cố định trong hình học.

Nhớ là dự đoán thôi.

Và cũng như vậy, trong 1 số bdt, để tìm đánh giá tốt nhất ta cũng dùng phương pháp này. okie?!


#39000 Bất đẳng thức thuần nhất

Gửi bởi MrMATH trong 22-10-2005 - 08:49

khi nào ta chọn số cho phương pháp chuẩn hóa cho tiện việc chứng minh

Có lẽ ý của em là ta nên chọn số như thế nào trong phép chuẩn hóa.

Thực ta thì đứng về mặt lý thuyết mà nói, việc chọn số để tính toán tiếp thực ra là tương đương nhau (theo ý là nếu dùng số này để đạt được kết quả thì cũng có thể dùng số khác để thu được đúng những kết quả đó)

Như vậy là, cách chọn số hoàn toàn mang tính chất thực hành và trực quan, chẳng hạn thế này, nếu có dạng căn bậc 2 thì chúng ta sẽ chọn những số có liên quan tới chính phương như 4,9,......... để trong quá trình tính toán, những con số mà chúng ta làm việc với càng gần số nguyên hay hữu tỷ là tốt nhất.

Anh nghĩ là, càng làm nhiều thì sẽ tự khắc thu được kinh nghiệm.

Châm ngôn của các bài toán sơ cấp (theo anh) là: chân lý và cái đẹp ở rất gần nhau.


#34739 Giới thiệu forum mới

Gửi bởi MrMATH trong 12-09-2005 - 17:02

Gửi các thành viên trẻ tuổi của Diễn đàn Toán học


Từ những năm 70 của thế kỷ trước, tạp chí Toán học và Tuổi trẻ đã đến với đông đảo bạn đọc trong nước và trở thành niềm tự hào của toán học trẻ Việt Nam. Tạp chí đã trở thành cầu nối cho các bạn trẻ yêu toán trên mọi miền đất nước, khơi dậy ở họ niềm đam mê toán học, và cũng từ đây, đã có một đội ngũ các nhà toán học trưởng thành, hiện đang công tác trong và ngoài nước. Gần đây, lại có một bước đột phá mới trong việc học và dạy toán trong nước. Đó chính là sự ra đời của 2 tạp chí toán học mới: Toán Tuổi Thơ I và II. Nếu như tạp chí Toán Tuổi Trẻ là nơi giao lưu chủ yếu của các bạn học sinh cuối cấp II và cấp III, thì 2 tạp chí Toán Tuổi Thơ I và II lại là vườn ươm của nhưng tài năng độ tuổi cấp I và đầu cấp II.

Một trong những mục tiêu của Diễn đàn Toán học là xây dựng một thế giới ảo dành cho các bạn trẻ yêu toán trao đổi về những vấn đề mình quan tâm. Rõ ràng, việc liên kết giữa Diễn đàn và các tạp chí nêu trên là rất có ích đối với các bạn học sinh yêu Toán trong và ngoài nước. Đối với các bạn trong nước, các bạn có thể trao đổi trực tuyến với nhau về nội dung của báo, đối với các bạn hiện ở nước ngoài, bạn có thể theo sát tình hình học và giảng dạy Toán học phổ thông trong nước, cũng như thảo luận với các bạn trong nước về những điều mình quan tâm.

Như các bạn đã biết, Toán học và Tuổi Trẻ chưa có diễn đàn chính thức, vì vậy diễn đàn đã tạo Một forum thảo luậnvề các bài toán được đăng tải trên tạp chí đó. Đây chính là một biểu hiện đầu tiên của Diễn đàn để tiến tới sự hợp tác đó.

Mặt khác, trong trường hợp các bạn đã biết tới tạp chí Toán Tuổi Thơ, thì hẳn đã tới thăm Trang chủDiễn đàn riêng của tạp chí đó. Vì vậy nhóm điều hành diễn đàn đã rất băn khoăn đến việc liệu có nên làm 1 việc tương tự như đối với tạp chí Toán Tuổi Trẻ trước đây hay không.

Sau 1 thời gian cân nhắc, chúng tôi cho rằng việc tạo ra 1 forum dành cho việc giới thiệu các chủ đề nóng trên Diễn đàn của tạp chí Toán Tuổi Thơ là rất cần thiết vì mấy nhẽ:

Thứ nhất là vì các thành viên của diễn đàn 3T rất trẻ, do đó các thảo luận chưa được sâu sắc. Hơn nữa các bạn đó rồi sẽ tiếp tục phát triển và sẽ có 1 lúc nào đó cần tới 1 diễn đàn dành cho các đối tượng có kiến thức cao hơn. Việc làm này sẽ tạo đà cho các bạn trẻ đó tiếp tục phát triển.

Thứ hai là việc làm này sẽ tập trung được đông đảo hơn đội ngũ các bạn trẻ yêu toán trong nước. Hi vọng đến một lúc nào đó giới trẻ Việt Nam yêu toán sẽ có một ngôi nhà chung thực sự. Để được như vậy, chúng ta nên bắt đầu từ ngày hôm nay với forum mới: Tạp chí Toán Tuổi Thơ

Chúc các bạn vui !

Nhóm điều hành diễn đàn.


#33800 Dựng hình chỉ bằng compa

Gửi bởi MrMATH trong 05-09-2005 - 10:06

90-60=30, 30-19=11,19-11=8,11X2-19=3,3X3-8=1

30-7X4=2,2X4=8,8-7=1


#32735 Bướm đơn bướm kép - Định lý Haruki

Gửi bởi MrMATH trong 27-08-2005 - 19:41

BƯỚM ĐƠN BƯỚM KÉP -ĐỊNH LÝ HARUKI




Trong thế giới toán học, đâu phải chỉ có sự ngự trị của các con số. Trong mục này các bạn sẽ thấy chúng - những con số - chỉ là phần tĩnh của những thực thể sống. Đó là cả 1 phiên bản đẹp của thế giới động vật đầy sức sống. Nắm được cái tĩnh trong động là 1 việc khó. Nhưng nắm được cái động trong tĩnh lại là 1 công việc đầy thú vị.............


Bài toán bướm đơn:
$PQ$ là 1 dây cung của đường tròn $(O)$; $AB, CD$ là 2 dây cung của đường tròn đó và cùng đi qua trung điểm $E$ của dây cung $PQ$; $AD,BC$ cắt $PQ$ tại $M,N$ tương ứng. Khi đó $MN$ cũng nhận $E$ làm trung điểm.
conbuom.PNG
Hình vẽ trên là 1 minh họa sống động cho định lý này: các bạn có thể thấy rõ phần gạch chéo là hình ảnh của 1 chú bướm đang xoè cánh bay. Và nếu như nối $AC,BD$ cắt $PQ$ thì ta lại nhận được 1 chú bướm khác.

Chứng minh định lý này như thế nào? Có rất nhiều cách: chẳng hạn lấy $C'D'$ đối xứng với CD qua trung trực của PQ rồi chứng mình tam giác C'EM và CEN bằng nhau. Hoặc chứng minh trực tiếp tam giác OMN cân.
buom2.PNG
Hầu hết những cách chứng minh hình học đều rất thú vị (không chỉ với những bài toán hình học mà ngay cả với những bài toán đại số cũng thế). Tuy nhiên dưới đây chúng ta sẽ làm quen với 1 chứng minh mang tính đại số hơn, bởi vì những hiệu quả của nó đem lại là rất đáng quan tâm.

Chứng minh của định lý bướm đơn: Gọi M',M" (tương ứng N',N") là hình chiếu vuông góc của M (tương ứng N) trên AB,CD. Lần lượt có:
  • $\dfrac{AM.MD}{CN.ND}=\dfrac{EM^2}{EN^2}$
  • $x=PM,x'=PM',y=MN,y'=M'N',z=NQ,z'=N'Q$

Lập luận như trên ta có: $\dfrac{y(x'-x)}{(z-z')y'}=\dfrac{x(y+z)}{(x'+y')z'}$ và từ đó $x.y'.z=z'.y.z'$

Hình đã gửi


Bây giờ nhờ định lý Haruki ta thu được chứng minh khá đơn giản cho bài toán bướm kép:

Bài toán bướm kép:Các dây cung đuợc bố trí như hình vẽ.Giả sử $PF=P'F',PG=P'G',PH=P'H'$

Dùng định lý Haruki ta có
  • $\dfrac{PE.FP'}{EF}=\dfrac{PG.HP'}{GH}$
  • $\dfrac{P'E'.F'P}{E'F'}=\dfrac{P'G'.H'P}{G'H'}$
Từ đó suy ra $PE=P'E'$

Hình đã gửi




#27023 Học toán cao cấp quá ...Để làm gì vậy ?

Gửi bởi MrMATH trong 11-07-2005 - 14:15

Hi, chào "con-meo", trước khi có đôi ý kiến replis, mình mạn phép hỏi bạn: "bạn sống để làm gì"
1 lúc nào, khi bạn có câu trả lời thỏa đáng thì lúc bấy giờ tôi cũng có thể có 1 kiến giải thích hợp cho câu hỏi đáng suy nghĩ này của bạn
Còn bây giờ xin mạn bàn 1 số đỉểm thế này
Tất nhiên trong cuộc sống có vô số người sống mà không cần học toán, nhưng có lẽ họ cũng có những lý tưởng của riêng mình . Tất nhiên có cả những người sống mà không cần lý tưởng, và họ vẫn sống được, nhưng chắc hẳn bạn không muốn có 1 cuộc sống như vậy. Vì thế toán học như 1 lý tưởng của các nhà toán học.
Một mặt khác, nhờ toán học mà các ngành khoa học khác có thể cùng phảt triển, và sự thật là từ toán học đã hình thành vô số ngành khoa học quan trọng đang ngày 1 giúp ích cho cuộc sống của con người, điều đó chẳng đáng để chúng ta học toán lắm sao
Cuối cùng,học toán như 1 nhu cầu không thể thiếu trong cuộc sống của 1 số người, và phần lớn những người đó đều trở thành những nhà toán học
Vài lời mạn bàn, không phiền hà gì, hi vọng có các cao kiến
okie?!



#14856 $g^{3}(G)\le{c\cdot{f^4(G)}}$

Gửi bởi MrMATH trong 03-04-2005 - 08:13

Tìm hằng số $C$ nhỏ nhất sao cho với mọi đồ thị hữu hạn $G$ ta có
$$g^{3}(G)\le{c\cdot{f^4(G)}}$$
trong đó $g(G)$ và $f(G)$ lần lượt là số các tứ diện, số các tam giác trong $G$




#14855 IMO-SL-04

Gửi bởi MrMATH trong 03-04-2005 - 08:12

Bài 7: Xác đinh tất cả các giá trị của $m, n$ sao cho hinh chữ nhật $m . n$ có thể lát khít kín bởi các hock:
**
*
***
  • LNH yêu thích


#14851 IMO-SL-04

Gửi bởi MrMATH trong 03-04-2005 - 08:09

Bài 3: đối với 1 đồ thị hữu hạn ta có thể xóa 1 cạnh tùy ý trong 1 vòng 4 cạnh tùy ý. Với đồ thị đầy đủ n đỉnh thì việc xóa cạnh có thể kết thúc sau ít nhất bao nhiêu lần?
  • LNH yêu thích


#14326 Ứng dụng của 1 đẳng thức đơn giản

Gửi bởi MrMATH trong 28-03-2005 - 16:49

Ứng dụng của 1 đẳng thức đơn giản
Trong bài viết này, tôi muốn trao đổi với các bạn về 2 bài toán sau:
Bài toán 1: Cho $n$ là số nguyên, $n>1$ , $a_1,a_2,..,a_n$ là $n$ số thực. Đặt $S=\sum_{i=1}^{n}{a_{i}^2}$, $P=\min_{i<j}{\left ( a_i-a_j\right )^2}$. Chứng minh rằng:
$$\dfrac{S}{P}\ge{\dfrac{n\left ( n-1\right )\left ( n+1\right )}{12}}.$$

Bài toán 2: Cho $n$ là số nguyên, $n>1$, $a_1,a_2,..,a_n$ là $n$ số thực. Chứng minh rằng tồn tại $n$ số thực $b_1,b_2,..,b_n$ thỏa mãn:
$ i) \forall{i=\overline{1,n}}$,$a_i-b_i\in\mathbb{Z};$
$ ii) \sum_{1\leq {i}<j\leq {n}}{\left ( b_i-b_j\right )^2}\leq {\dfrac{n^2-1}{12}}.$

Hai bài toán này có mối liên hệ với nhau như thế nào? Dù chưa giải, các bạn cũng có thể thấy ngay sự giống nhau của 2 BĐT là ở nhân tử chung $\dfrac{n^2-1}{12}$, nhưng bài toán 2 có vẻ khó hơn rất nhiều so với bài toán 1, vì thế chúng ta sẽ thử giải bài toán 1 trước. Yêu cầu của bài toán khiến ta có cảm giác có thể cố định 1 trong 2 đại lượng $S,P$. Rõ ràng nếu phải chọn 1 trong 2 đại lượng đồng bậc đó thì ta sẽ chọn $P=1$.

Nhận xét: Với: $0<b_1<b_2<..<b_k,k>1,b_{i+1}-b_{i}\ge{1}$, $\forall i=\overline{1,k}$ thì
$$\sum_{i=1}^{k}b_{i}^2\ge \sum_{i=1}^{k}{\left ( b_1+i-1\right )^2}= k b_{1}^2+k\left ( k-1\right )b_1+\dfrac{2k^3-3k^2+k}{6} \textbf{ (1)}.$$
Hơn nữa $\left ( 1\right )$ sẽ trở thành đẳng thức nếu như $k=0$ hoặc $k=1$, nên $\left ( 1\right )$ đúng với mọi $n$ tự nhiên.

Áp dụng: Giả sử
$$a_1<a_2<...<a_p=-\alpha\le{0}\le{\beta}=a_{p+1}=...=a_n.$$
trong đó
$$p\geq{0},q=n-p\geq{0},\alpha\geq{0},\beta\geq{0},\alpha+\beta\geq{1}$$,
có thể giả thiết $p\geq{q}$ (vì nếu không ta xét bộ số $\left ( -a_n,..,a_1\right )$), như thế $p\geq{1}$.
Áp dụng $\left ( 1\right )$ cho 2 bộ số $\left ( -a_p,..,-a_1\right ),\left ( a_{p+1},..,a_n\right )$, dẫn tới:
$$ S\geq p\alpha^2+q\beta^2+p\left ( p-1\right )\alpha+q\left ( q-1\right )\beta+C=f\left ( \alpha\right )$$,
trong đó:
$$C=\dfrac{1}{6}.\left [ 2\left ( p^3+q^3\right )-3\left ( p^2+q^2\right )+p+q\right ]$$.
Do đó:
$$f'\left ( \alpha\right )=2p\alpha+p\left ( p-1\right )\geq{0},\forall{\alpha\ge{0}}$$.
Vì thế $f\left ( \alpha\right )\geq {f\left ( 1-\beta\right )}$, đặt $b=\left ( \beta-1\right )-\left ( p-1\right )$ thì:
$$S\geq \sum_{i=1}^{n} \left ( b+i-1\right )^2 =f\left ( b\right )$$.
Dễ thấy $f\left ( b\right )$ là 1 tam thức biến $b$, hệ số cao nhất là $n$, hệ số của $b$ là $n-1$, do đó:
$$f\left ( b\right )\geq f\left ( -\dfrac{n-1}{2}\right )=\dfrac{n\left ( n-1\right )\left ( n+1\right )}{12}$$.
Bài toán đã được giải xong.
Trong 1 thời gian dài, tôi rất hài lòng với cách giải này, vì nó rất logic, cho đến bây giờ tôi vẫn nghĩ thế. Có điều với cách giải này thì khó lòng mà tiến công được bài toán 2.

Nhưng làm toán cũng có nhiều sự may mắn, ngẫu nhiên tôi bắt gặp đẳng thức:
$$\sum_{i<j}\left ( a_i-a_j\right )^2 = n\sum_{i=1}^{n} a_{i}^2 -\left(\sum_{i=1}^{n}a_i\right)^2\textbf{ (*)}$$
Nhờ đẳng thức này mà bt1 có thể giải rất ngắn gọn như sau:

Giải bài toán 1:
Giả sử: $a_1<a_2<...<a_n$ và: $\mathop\min\limits_{1\leq{i}<n} \left ( a_{i+1}-a_i\right )=\sqrt{P}$.
$$\Rightarrow \forall{j>i},a_j-a_i\geq \left ( j-i\right )\sqrt{P} \Rightarrow \left ( a_j-a_i\right )^2\geq \left ( j-i\right )^2P$$,
Cộng hết lại suy ra:
$$nS\ge n\sum_{i=1}^{n} a_{i}^2 -\left ( \sum_{i=1}^{n} a_i \right )^2=\sum_{i<j} \left ( a_i-a_j\right )^2 \geq P\sum_{i<j} \left ( j-i\right )^2$$.
$$\Rightarrow \dfrac{S}{P}\ge \dfrac{n\left ( n-1\right )\left ( n+1\right )}{12}$$.
Thừa thắng xông lên, liệu nhờ đẳng thức này bài toán 2 có được giải hay không? Câu trả lời là có, nhưng vấn đề không đơn giản quá!

Giải bài toán 2:
$B=\{\left ( b_1,b_2,..,b_n\right )| b_i-a_i\in\mathbb{Z}\}, \forall i=\overline{1,n}$ và $F=\{f\left ( S\right )|S\in{B}\}$ có phần tử nhỏ nhất, giả sử đó là $f\left ( S\right )$ trong đó $s_0=0$, và dùng BĐT Bunhiacopxki ta có:
$$ \sum_{i=1}^{n} s_{i}^2.\sum_{i=1}^{n} \left ( \dfrac{i}{n}\right )^2 \geq \left ( \sum_{i=1}^{n} \dfrac{i}{n}\right )^2 =\left [ \sum_{i=1}^{n} ( s_i-s_{i-1}). \sum_{j=i}^{n} \dfrac{j}{n}\right ]^2$$.
$$\geq \left [ \sum_{i=1}^{n}( s_i-s_{i-1}). \sum_{j=i}^{n} s_j\right ]^2 = \left ( \sum_{i=1}^{n} s_{i}^2\right )^2$$.
$$\Rightarrow \sum_{i=1}^{n} s_{i}^2 \le \sum_{i=1}^{n} \left ( \dfrac{i}{n}\right )^2 $$.
Cuối cùng, dùng $(*)$, ta có:
$$f(S)= n\sum_{i=1}^{n} s_{i}^2 -\left ( \sum_{i=1}^{n} s_i\right )^2\leq n\sum_{i=1}^{n} \left ( \dfrac{i}{n}\right )^2 -\left ( \dfrac{n+1}{2}\right )^2 =\dfrac{n^2-1}{12}$$.
Bài toán đã được giải xong.

Có thể thấy Bài toán 2 đã được giải khá đẹp nhờ $(*)$, có điều không chỉ là đem lại thắng lợi lớn như vậy, mà trong chứng minh thực ra đã sử dụng tới công thức biến đổi Abel, phép biến đổi Abel có nội dung như sau:

$$\sum_{i=1}^{n} a_ib_i =\sum_{i=1}^{n} \left ( a_i-a_{i+1}\right )\left ( \sum_{j=1}^{i} b_j \right ) $$,
Ở đây $a_{n+1}=0$
Chi tiết về phép biến đổi Abel và những ứng dụng của nó, xin phép được trao đổi với các bạn trong 1 dịp khác


#14216 phương pháp cân bằng hệ số trong chứng minh bđt

Gửi bởi MrMATH trong 27-03-2005 - 15:32

Phương pháp cân bằng hệ số trong chứng minh bất đẳng thức


Chúng ta bắt đầu từ 1 ví dụ nhỏ:
VD1: tìm giá trị lớn nhất của biểu thức $x^3(1-x^2)$, trong đó $x\ge{0}$
Bài toán có thể giải bằng phương pháp khảo sát hàm số khá dễ dàng, tuy nhiên chúng ta sẽ đi theo 1 hướng suy nghĩ khác để tìm cách giải sơ cấp hơn.
Yêu cầu của bài toán là tìm giá trị lớn nhất của 1 biểu thức là tích của 5 đơn thức biến x, đó là $\dfrac{1}{ab}x\cdot{x}\cdot{x}\cdot{(a-ax)}\cdot{(b+bx)}$.
Điều kiện của a,b là a,b là các số thực dương, $b+3=a$,$\exists{x\in{[0,1]}},x=a-ax=b+bx$, giải hệ ta có $a=\dfrac{3+\sqrt{15}}{2},b=\dfrac{\sqrt{15}-3}{2}$, từ đó $ab=\dfrac{3}{2}$, như vậy
$x^3(1-x^2)=\dfrac{2}{3}\cdot{[\dfrac{3+\sqrt{15}}{2}(1-x)]}\cdot{[\dfrac{\sqrt{15}-3}{2}(1+x)]}\le{\dfrac{2}{3}(\dfrac{\sqrt{15}}{5})^5}=\dfrac{6\sqrt{15}}{125}$, đẳng thức xảy ra $\Leftrightarrow{x=\dfrac{\sqrt{15}}{5}}$
Qua ví dụ nhỏ này ta thấy việc sử dụng BĐT Cốsi (nếu dùng được) sẽ quy về việc giải 1 hệ phương trình, và trên lý thuyết thì có thể chuyển hệ đó về 1 PT1 biến bậc cao, nhưng cho dù Pt đó có nghiệm thì cũng chưa chắc đã tìm được nghiệm đó, đây chính là điểm hạn chế của PP này.
Để hiểu rõ hơn những kết luận này, xin mời các bạn giải quyết vài bài toán sau:
BT1: tìm giá trị nhỏ nhất của biểu thức $a(x^2+y^2)+z^2$ trong đó a là hằng số, x,y,z là các biến thỏa mãn: $xy+yz+zx=1$
BT2: giả sủ x là số lớn nhất trong 3 số x,y,z. tìm giá trị nhỏ nhất của biểu thức : $\dfrac{x}{y}+\sqrt{1+\dfrac{y}{z}}+\sqrt[3]{1+\dfrac{z}{x}}$
BT3: tìm giá trị lớn nhất của các biểu thức 1 biến a sau:
1/$T_{2,1}=a(m-a)(n-a)$ trong đó $0<a<\min{(m,n)}$
2/$T_{3,3}=a^3(m-a)(n-a)(p-a)$ trong đó $0<a<\min{(m,n,p)}$
3/$T_{k,n}=a^{n}\cdot{\pi_{i=1}^{k}{(t_i-a)}}$ trong đó $0<a<\min_{1\le{i}\le{k}}{t_i}$
BT4: xét các số thực dương a,b,c thỏa mãn: $21ab+2bc+8ca\le{12}$, tìm giá trị nhỏ nhất của biểu thức: $\dfrac{1}{a}+\dfrac{2}{b}+\dfrac{3}{c}$
Chúng ta biết rằng dùng BĐT Cốsi có thể CM BĐT Bunhiacopxki, và ngược lại, do đó trên nguyên tắc 2 BĐT này là tương đương với nhau, nhưng trong kỹ thuật cân bằng hệ số thì chính dạng tồn tai của 2 BĐT đã tạo nên nhưng đặc thù, mà chúng ta sẽ theo dõi qua vài ví dụ dưới đây:
BĐT Bunhiacopxki có nội dung như sau: $\Large(\sum_{i=1}^{k}{x_i})^2\le{(\sum_{i=1}^{k}{c_i})(\sum_{i=1}^{k}{\dfrac{x_{i}^2}{c_i}})}$:P, trong đó $x_i$ là các số thực được coi là các biến tự do, còn $c_{i}\neq{0}$ được coi như những hằng số.
VD2:với mỗi số nguyên dương n cho trước, tìm số $t=t(n)$ nhỏ nhất sao cho $\forall{x_{i}\in{R}},\forall{i=\overline{1,n}}$
ta có :
$\sum_{k=1}^{n}{(\sum_{i=1}^{k}{x_i})^2}\le{t\cdot{\sum_{i=1}^{n}{x_{i}^2}}}$
Giải: Áp dụng :P cho $k=\overline{1,n}$ rồi cộng lại ta có:
$\sum_{k=1}^{n}{(\sum_{i=1}^{k}{x_i})^2}\le{\sum_{k=1}^{n}{S_{k}(\sum_{i=1}^{k}{\dfrac{x_{i}^2}{c_i}})}}$ trong đó: $S_k=\sum_{i=1}^{k}{c_i},\forall{k=\overline{1,n}}$
Đến đây ta phải chọn $c_k$ sao cho:$\dfrac{S_1+S_2+...+S_n}{c_1}=\dfrac{S_2+..+S_n}{c_2}=...=\dfrac{S_n}{c_n}=t$, nhưng phải chọn thế nào đây? Các bạn thử suy nghĩ xem cách chọn sau có ngẫu nhiên hay không:
$\forall{i=\overline{1,n}},c_i=\sin{i\alpha}-\sin{(i-1)\alpha}$ trong đó: $\alpha=\dfrac{\pi}{2n+1}$ khi đó $t=\dfrac{1}{4\sin^2{(\dfrac{\alpha}{2})}}$
Qua ví dụ này ta thấy việc sử dụng BĐT Bunhiacopxki cũng có thể quy về 1 hệ đẳng thức, nhưng do điều kiện đẳng thức thường dễ tìm hơn BĐT Cốsi nên việc giải hệ ít khi đưa về PT bậc cao mà lại yêu cầu 1 vốn kiến thức tổng hợp, nói nôm na là phải biết chọn đẳng thức, bất đẳng thức phụ thích hợp cho từng trường hợp.
VD3: chứng minh $(\sum_{i=1}^{n}{x_i})^2<\dfrac{\pi^2}{6}\cdot{\sum_{i=1}^{n}{i^2x_{i}^2}}$, hơn nữa: $\dfrac{\pi^2}{6}$ là hằng số không thể làm bé hơn
Nhận xét:ta sử dụng (*) với $c_i=\dfrac{1}{i^2},\forall{i=\overline{1,n}}$ và dùng định lý euler: $\lim_{n\to{\infty}}{\sum_{i=1}^{n}{\dfrac{1}{i^2}}=\dfrac{\pi^2}{6}$
VD4: chứng minh: $(\sum_{i=1}^{n}{x_i})^4\le{\pi^2\cdot{(\sum_{i=1}^{n}{x_{i}^2})(\sum_{i=1}^{n}{i^2x_{i}^2})}$
Giải: ta chọn $c_i=\dfrac{1}{t+\dfrac{i^2}{t}}$,$t>0$ là 1 hằng số sẽ được chọn sau. Vậy thì $(\sum_{i=1}^{n}{x_i})^2\le{S_{n}\cdot{(tP+\dfrac{Q}{t})}}$ với $OM_0=t$, trên tia $M_{0}M_{1}=...=M_{n-1}M_{n}=1$, và $\forall{i=\overline{1,n}}$ đặt $\widehat{M_{i-1}OM_{n}}=\alpha_i$. Tính diện tích $K_i$ của $\Delta{M_{i-1}OM_{n}}$ theo 2cách:$t=\sqrt{\dfrac{Q}{P}}$ ta nhận được điều phải chứng minh
Phụ lục A: về các bất đẳng thức Carlson:2 BĐT được đề cập đến trong CD3, VD4 đều có tên chung là các BĐT Carlson, dễ thấy rằng mỗi BĐT Carlson cần đến 1 phương án giải quyết khác nhau, mặc dù kỹ thuật của chúng đều là cân bằng hệ số. Để hiểu rõ hơn về vấn đề này mời các bạn thử giải bài toán sau
BT: các BĐT sau có đúng không, nếu đúng hãy chứng minh, có thẻ làm chặt hơn không, nếu có thể hãy chỉ ra đánh giá tốt nhất:
i) $(\sum_{i=1}^{n}{x_i})^4<54(\sum_{i=1}^{n}{x_{i}^2})(\sum_{i=1}^{n}{i\cdot{x_{i}^2}})(\sum_{i=1}^{n}{i^2x_{i}^2})$
ii)$(\sum_{i=1}^{n}{x_i})^{15}<3\cdot{10^5}(\sum_{i=1}^{n}{x_{i}^3})(\sum_{i=1}^{n}{i\cdot{x_{i}^3}})(\sum_{i=1}^{n}{i^2x_{i}^3})(\sum_{i=1}^{n}{i^3x_{i}^3})(\sum_{i=1}^{n}{i^3x_{i}^3})(\sum_{i=1}^{n}{i^4x_{i}^3})$
Phụ lục B: chứng minh định lý euler:$\lim_{n\to{\infty}}{\sum_{i=1}^{n}{\dfrac{1}{i^2}}=\dfrac{\pi^2}{6}$:
Chúng ta đã biết công thức Moivre: $\cos{nx}+i\sin{nx}=(\cos{x}+i\sin{x})^n$, cân bằng hệ số phần ảo rồi thay $n:=2n+1$ ta có
$\sin{(2n+1)x}=\sin^{2n+1}{x}(C^{1}_{2n+1}cotg^{2n}{x}-C^{3}_{2n+1}cotg^{2n-2}{x}........)$
Như thế n số $cotg^{2}{\dfrac{k\pi}{2n+1}},k=\overline{1,n}$ là tất cả các nghiệm của đa thức bậc n: $C^{1}_{2n+1}X^N-C^{3}_{2n+1}X^{n-1}..........$, như thế áp dụng định lý Viet ta có $\sum_{i=1}^{n}{cotg^2{\dfrac{k\pi}{2n+1}=\dfrac{C^{3}_{2n+1}}{C^{1}_{2n+1}}=\dfrac{n(2n-1)}{3}\Rightarrow{\sum_{i=1}^{n}{\dfrac{1}{\sin^2{\dfrac{k\pi}{2n+1}}}}=\dfrac{2n(n+1)}{3}$, áp dụng bất đẳng thức kép: $\sin{\alpha}<\alpha<tg{\alpha},\forall{\alpha\in{(0,\dfrac{\pi}{2})}}\Rightarrow{\dfrac{1}{\sin{\alpha}}>\dfrac{1}{\alpha}>cotg{\alpha}$
Dẫn tới: $\dfrac{\pi^2}{6}\cdot{[(1-\dfrac{2}{2n+1})(1-\dfrac{1}{2n+1})]<\sum_{i=1}^{n}{\dfrac{1}{i^2}}<\dfrac{\pi^2}{6}\cdot{[(1+\dfrac{1}{2n+1})(1-\dfrac{1}{2n+1})]$, lấy lim 2 vế, áp dụng nguyên lý kẹp, ta có điều phải chứng minh
Tài liệu tham khảo: báo toán học và tuổi trẻ, tạp chí kvant số 6 năm 1974



#12964 CM trong 1 tập có chứa ít nhất 3 phần tử(không nhất thiết phân biệt) thỏa mãn...

Gửi bởi MrMATH trong 18-03-2005 - 18:17

Đây là 1 trường hợp của bài toán SCHUR cho tập hợp
Hint: có thể dùng tô màu, hoặc có thể đếm thông thường
  • LNH yêu thích


#12599 Món quà từ thế giới cổ đại

Gửi bởi MrMATH trong 16-03-2005 - 15:03

MÓN QUÀ TỪ THẾ GIỚI CỔ ĐẠI

Tổ tiên của chúng ta-những người cổ đại-đã để lại 7 kì quan quý giá: những kim tự tháp Ai Cập, tượng thần Zeus Hi Lạp, lăng Halicanase, miếu thờ thần Artemis, vườn treo Babylon, tượng thần khổng lồ Rhodes và cuối cùng là ngọn hải đăng Alexandria.

Có khi nào bạn tự hỏi mình rằng "bằng cách nào mà họ- những con người nhỏ bé- lại có thể sáng tạo ra những tác phẩm kì vĩ đến như vậy?".

Bảy kì quan đó rất có thể sẽ bị tàn phá bởi sự khắc nghiệt của thời giạn Nhưng chiếc chìa khoá để tạo nên những điều kì diệu đã được truyền lại cho chúng ta, đó chính là kì quan thứ 8 của thế giới cổ đại: Toán Học.

Bạn có thể cho rằng toán học chỉ đơn thuần là những con số khô khan còn chính tính thẩm mĩ mới tạo ra vẻ đẹp của các kì quan thế giới. Điều đó đúng quá, nhưng có đúng hoàn toàn không?

Chúng ta hãy cùng làm một thí nghiệm nhỏ: giả sử bạn đã từng chiêm ngưỡng vẻ đẹp của điện Parteông, và có lẽ bạn cho rằng chính sự cân đối đã tạo nên vẻ đẹp đó (trong trường hợp nó đẹp đối với bạn), hơn nữa lại giả sử bạn chưa biết rằng tỷ lệ chiều rộng và chiều cao của mặt điện là xấp xỉ 1,5. Bây giờ nhờ bạn vẽ thật nhanh một hình chữ nhật vừa mắt bạn nhật. Không biết bạn sẽ nghĩ sao nếu như tỷ lệ chiều dài và chiều rộng của hình chữ nhật bạn vừa vẽ cũng ở khoảng 1,5.
Những thí nghiệm chặt chẽ hơn cho thấy con số chính xác hơn 1,5 lại là 1,618. Trong đời sống người ta gọi đây là con số vàng, vì sao vậy, bởi vì người ta đã bắt gặp nó trong rất nhiều thứ được cho là đẹp.

Vậy có thể gọi đây là một hằng số đẹp dược hay không Tất nhiên là được, bởi vì đó chỉ là vấn đề qui ước.Và do đó chừng nào còn các hằng số đẹp thứ 2, thứ 3 thì chừng ấy những cách làm cho một vật trở nên đẹp đối với chúng ta còn vô cùng đa dạng.

Với một thí dụ nhỏ như vậy tôi không hi vọng các bạn có thể chấp nhận:" mọi con số đều có linh hồn của nó, và thế giới của các con số có khi còn phức tạp hơn thế giới loài người (không chỉ vì nó đông dân hơn)", và đương nhiên tôi lại càng không thể ép các bạn công nhận điều đó.Trái lại, có lẽ các bạn sẽ đánh giá được đúng mức độ của nó trong quá trình đi tìm phản bác cho chính quan niệm chủ quan đó.
Âu đó cũng là một phong cách tư duy toán hoc.

Đêcac đã triết lí rất hay:"tôi tư duy tức là tôi tồn tại", mặt khác:"toán học lại là một hình thức tư duy tự nguyện- ¬William James-", vậy:"tôi làm toán nghĩa là tôi tồn tại".

Tôi cho rằng chiều ngược lại của mệnh đề này cũng đúng, còn bạn.....


#10692 Những Con Số Thú Vị

Gửi bởi MrMATH trong 03-03-2005 - 17:28

có 1 số rất đẹp: 854 hãy bình phương nó lên, bạn sẽ thấy đủ các chữ số còn lại.okie?
còn 1 số như thế này nữa: ai có thể tìm ra đầu tiên đây?


#10498 $\sqrt[3]{ab}+\sqrt[3]{cd}\leq {...

Gửi bởi MrMATH trong 02-03-2005 - 16:28

Bài toán: Cho 4 số dương $a,b,c,d$. Chứng minh bất đẳng thức:
$$\sqrt[3]{ab}+\sqrt[3]{cd}\leq {\sqrt[3]{(a+b+c)\cdot({b+c+d})}}$$